For each ordered pair, determine whether it is a solution to y=-9.
Is it a solution?
Yes or No
(1, -9)
(7,3)
(-9,4)
(0, -9)

Answers

Answer 1

Answer:

(1, -9)  yes

(7,3)  no

(-9,4)  no

(0, -9) yes

Step-by-step explanation:

The y value must be -9

The x value can be any value to satisfy   the equation y = -9


Related Questions

Can someone please help me? Please C: And thanks!

How much would you need to deposit in an account each month in order to have $50,000 in the account in 8 years? Assume the account earns 4% annual interest compounded monthly.

Answers

Answer:

$540.98

Step-by-step explanation:

future value= $ 50,000

number of deposits (n)= 8*12 = 96

rate (r) = 4% per month

= 4÷12 per annum

= 0.33% p.a

i = 0.33÷100

= 0.0033

We know,

Future value of annuity = P÷i [ (1 + i)^n - 1 ]

$50,000 = P÷ 0.0033 [ ( 1+0.0033)^96 - 1]

$50,000 * 0.0033=P [ (1.0033)^96 - 1 ]

$165 = P*0.305

P = $165÷0.305

P = $ 540.98

Rough::

let x= 1.0033)^96

log x = 96 * log (1.0033)

log x = 0.1156

x = Antilog (0.1156)

= 1.305

1.305 - 1 = 0.305

Matrix A is said to be involutory if A2 = I. Prove that a square matrix A is both orthogonal and involutory if and only if A is symmetric.

Answers

Answer:

4 · 1/4 (I-0) = (A-0)∧2

see details in the graph

Step-by-step explanation:

Matrix A is expressed in the form A∧2=I

To proof that Matrix A is both orthogonal and involutory, if and only if A is symmetric is shown by re-expressing that

A∧2=I in the standard form

4 · 1/4 (I-0) = (A-0)∧2

Re-expressing

A∧2 = I as a graphical element plotted on the graph

X∧2=I

The orthogonality is shown in the graphical plot displayed in the picture. Orthogonality expresses the mutually independent form of two vectors expressed in their perpendicularity.

a mens clothing sore sold out of $50 jackets and $30 jackets for a total of $2360 if the store sold 12 more$30 jackets than$50 jackets how many$50 jackets were sold

Answers

Answer:

25

Step-by-step explanation:

Let x represent the number of $50 jackets that were sold, and let y represent how many $30 jackets were sold.

50x + 30y = 2360

y = x + 12

Solve by substitution by substituting the second equation into the first one. Then, solve for x:

50x + 30y = 2360

50x + 30(x + 12) = 2360

50x + 30x + 360 = 2360

80x + 360 = 2360

80x = 2000

x = 25

So, 25 $50 jackets were sold.

A certain game involves tossing 3 fair coins, and it pays .14 for 3 heads, .06 for 2 heads, and .01 for 1 head. The expected winnings are?

Answers

Answer:

Total expected amount = $0.04375

Step-by-step explanation:

We need to calculate probability of getting heads on every combination of coin tosses

HHH = 1/8 = 3 heads

HHT = 1/8 = 2 heads

HTH = 1/8 = 2 heads

HTT = 1/8 = 1 head

THH = 1/8 = 2 heads

THT = 1/8 = 1 head

TTH = 1/8 = 1 head

TTT = 1/8 = 0 head

So the probability of 3 heads is 1/8 and the amount is (1/8)* 0.14 = $0.0175

Probability of 2 heads is 3/8 and the amount is (3/8) * 0.06 = $0.0225

Probability of 1 head is 3/8 and amount is (3/8) * 0.01 = $0.00375

Total expected amount = 0.00375 + 0.0225 + 0.0175

Total expected amount = $0.04375

What is the value of x when y equals 66?

y=0.985897x+0.194185

Answers

Answer:

x = 66.74715005725

Step-by-step explanation:

First you bring over the added variable. 0.194185, and subtract it from 66. Then you divide your difference by 0.985897. This gives you 66.74715005725

I have this question on an assignment and my calculator won't show the horizontal asymptote correctly can I get some help here?

Answers

What's the question? I can try and help..

PLEASE HELP QUICK!!!Suppose the bill for dinner is $16.70, if you want to give a 10% tip what will be the total?

Answers

Answer:

$18.37

Step-by-step explanation:

$16.70 × 1.10 = $18.37

or

$16.70 × 0.10 = $1.67

$16.70 + 1.67 = $18.37

Please help! Determine whether the conjecture is true or false and put an example on why it is

Answers

Answer:

Step-by-step explanation:

The first one is true. There can't be any other choice.

a = 5959599949 b = 0 then a*b = 0 because b = 0

The Second one is also true, although you may stall trying to figure out what is meant.

Suppose the angle to start with is 30 degrees

There are two angles that are supplementary to this angle. They can only be 180 - 30 = 150 each. Therefore they are equal to each other. This happens because supplementary angles must add to 180 and nothing else.

The third one is false. You can think of states like Montana which has 3 syllables and Wyoming which also has 3. Texas has two. But guess what? Maine only has 1.

The last one is also false. If you square an even number, you get an even number. Add 1 and you get an odd number. 4^2 = 16 Add 1 you get 17. Seventeen is odd.

What are m and b in the linear equation, using the common meanings of m and b? 2 + 3x + 5 - 2x = y

Answers

y=mx+b is the general formula of linear equation

y=-2x+5+3x+2

y=1x+7

m=1

b=7

Linear equation given in the question is,

2 + 3x + 5 - 2x = y

To simplify this equation further,

Add like terms of the equation,

        (2 + 5) + (3x - 2x) = y

         7 + x = y

Now compare this linear equation with the slope-intercept form of the linear equation,

y = mx + b

Here, m = slope of the line'

b = y-intercept

By comparing the equations,

m = 1

b = 7

Learn more,

https://brainly.com/question/15253236

What would the 60 is x% of 12. Find the value of x.

Answers

Answer:

The value of x= 20

Step-by-step explanation:

I believe the question is ,"60% of x is us, find x"

So , if the percentage of x to 60 is 12.

60/100 * x = 12

0.6 *x = 12

Dividing both sides by 0.6

X= 12/0.6

X= (12/6) *(10)

X= 2*10

.x= 20

The value of x= 20

Help pleaseeeeeeeeeeeeeeeeeeee

Answers

Answer:

{-3, 1, 5, 6}

Step-by-step explanation:

The domain of a relation is the x-values represented in that function. In a coordinate pair, the x-value comes first, so all of the first numbers in each of the pairs are part of the domain. When writing domain, it should always be in the least to greatest order. Therefore, the domain is {-3, 1, 5, 6}. Since this relation is a function, none of the x-values will repeat.

Solve for h.
H+6/4= 5

Answers

H + 6/4 = 5
- 6/4 -6/4

H = 3.5 OR 3 1/2

Select the correct answer from each drop-down menu.
The function f is given by the table of values as shown below.

x 1 2 3 4 5
f(x) 13 19 37 91 253
Use the given table to complete the statements.

The parent function of the function represented in the table is
.

If function f was translated down 4 units, the
-values would be
.

A point in the table for the transformed function would be
.

Answers

Answer:

3^x9, 15, 33, 87, 249(4, 87) for example

Step-by-step explanation:

a) First differences of the f(x) values in the table are ...

  19 -13 = 6, 37 -19 = 18, 91 -37 = 54, 253 -91 = 162

The second differences are not constant:

  18 -6 = 12, 54 -18 = 36, 162 -54 = 108

But, we notice that both the first and second differences have a common ratio. This is characteristic of an exponential function. The common ratio is 18/6 = 3, so the parent function is 3^x.

__

b) Translating a function down 4 units subtracts 4 from each y-value. The values of f(x) in the table would be ...

  9, 15, 33, 87, 249

__

c) The x-values of the function stay the same for a vertical translation, so the points in the table of the transformed function are ...

  (x, f(x)) = (1, 9), (2, 15), (3, 33), (4, 87), (5, 249)

Answer: I think this is it:

The parent function of the function represented in the table is exponential. If function f was translated down 4 units, the f(x)-values would be decreased by 4. A point in the table for the transformed function would be (4,87)

Step-by-step explanation: I got it right on Edmentum!

HELP UUUURRRRRRRGGGGGEEEEEENNNNTTTTT PLLLLZZZZZ IM BAD AT MATHHHHHHHH

Answers

Answer:

-1 8/9

Step-by-step explanation:

w + ( - x)

w = -5/9

z = 4/3

Input:

-5/9 + ( -4/3)

-5/9 - 4/3

-4/3 * 3/3 = -12/9

-5/9 - 12/9 = -17/9 = -1 8/9

If my answer is incorrect, pls correct me!

If you like my answer and explanation, mark me as brainliest!

-Chetan K

Find the area of the parallelogram with vertices A(−1,2,3), B(0,4,6), C(1,1,2), and D(2,3,5).

Answers

Answer:

5*sqrt3

Step-by-step explanation:

The vector AB= (0-(-1), 4-2,6-3) AB= (1,2,3)

The modul of AB is sqrt(1^2+2^2+3^2)= sqrt14

The vector AC is (1-(-1), 1-2, 2-3)= (2,-1,-1)

The modul of B is sqrt (2^2+(-1)^2+(-1)^2)= sqrt6

AB*AC= modul AB*modul AC*cosA

cosA=( 1*2+2*(-1)+3*(-1))/ sqrt14*sqrt6= -3/sqrt84=

sinB= sqrt (1- (-3/sqrt84)^2)= sqrt75/84= sqrt 25/28= 5/sqrt28

s= modul AB*modul AC*sinA= sqrt14*sqrt6* 5/ sqrt28= 5*sqrt3

Given there are 26 alphabets in the English language, how many possible three-letter words are there?

Answers

Answer: 17,576

We have 26 letters and 3 slots to fill. We can reuse a letter if it has been picked, so we have 26^3 = 26*26*26 = 17,576 different three letter "words". I put that in quotes because a lot of the words aren't actual words, but more just a sequence of letters.

what should be added to 66.778 get 78.2​

Answers

Answer:

11.422

Step-by-step explanation:

[tex]78.2 - 66.778 \\ = 11.422[/tex]

(Small sample confidence intervals for a population mean) suppose you are taking a sampling of 15 measurements. you find that x=75 and s =5. assuming normality, the 99% confidence interval for the population mean is:__________

Answers

Answer:

The 99% confidence interval is  [tex]71.67 < \mu < 78.33[/tex]

Step-by-step explanation:

From the question we are told that

     The sample  size  is  [tex]n = 15[/tex]

      The  sample  mean is  [tex]\= x = 75[/tex]

        The  standard deviation is  [tex]s = 5[/tex]

 Given that confidence is  99%  then the level of significance is mathematically represented as

              [tex]\alpha = 100 - 99[/tex]

             [tex]\alpha = 1\%[/tex]

             [tex]\alpha = 0.01[/tex]

Next we obtain the critical values of  [tex]\frac{ \alpha }{2}[/tex] from the normal distribution table

   The  value is

                  [tex]Z_{\frac{ \alpha }{2} } = 2.58[/tex]

Generally the margin for error is mathematically represented as

            [tex]E = Z_{\frac{ \alpha }{2} } * \frac{ s}{ \sqrt{n} }[/tex]

=>         [tex]E = 2.58 * \frac{ 5}{ \sqrt{15} }[/tex]

=>         [tex]E = 3.3307[/tex]

   The  99% confidence interval is mathematically represented as

             [tex]\= x -E < \mu < \= x +E[/tex]

=>          [tex]75 - 3.3307 < \mu <75 + 3.3307[/tex]

=>          [tex]71.67 < \mu < 78.33[/tex]


What is the midline equation of the function h(x) = -4 cos(5x - 9) - 7?

Answers

Answer: Midline equation: y = -7

Step-by-step explanation: This function is a sinusoidal function of the form:

y = a.cos(b(x+c))+d

Midline is a horizontal line where the function oscillates above and below.

In the sinusoidal function d represents its vertical shift. Midline is not influenced by any other value except vertical shift. For that reason,

Midline, for the function: [tex]h(x) = -4cos(5x-9) - 7[/tex] is y=d, i.e., [tex]y=-7[/tex]

Answer:

y=-7

Step-by-step explanation:

Henry takes out a $650 discounted loan with a simple interest rate of 12% for a period of 7 months. How much money does Henry receive into his bank account when the loan is drawn down? Give your answer to the nearest cent.

Answers

Answer:

$546

Step-by-step explanation:

Given

Amount, P = $650

Rate, R = 12%

Period, T = 7 months

Required

Determine the amount paid.

We'll solve this using simple interest formula, as thus

[tex]I = \frac{PRT}{100}[/tex]

Substitute values for T, R and P

[tex]I = \frac{\$650 * 12 * 7}{100}[/tex]

[tex]I = \frac{\$54600}{100}[/tex]

[tex]I = \$546[/tex]

Hence, Henry's withdrawal is $546

The area of a triangle is 24 square inches. What is the height of the triangle if the base length is 4 inches?
6 inches
8 inches
12 inches
20 inches

Answers

Answer:

[tex]\boxed {\boxed { \sf 12 \ inches}}[/tex]

Step-by-step explanation:

The area of a triangle can be calculated using the following formula.

[tex]a=\frac{1}{2} bh[/tex]

The area of the triangle is 24 square inches and the base is 4 inches long.

a= 24 in² b= 4 in

Substitute the values into the formula.

[tex]24 \ in^2 = \frac {1}{2} * 4 \ in * h[/tex]

Multiply on the right side of the equation.

[tex]24 \ in ^2 = ( \frac{1}{2} * 4 \ in ) * h[/tex]

[tex]24 \ in ^2 =2 \ in *h[/tex]

We are solving for the height of the triangle, so we must isolate the variable h. It is being multiplied by 2 inches. The inverse of multiplication is division, so we divide both sides by 2 inches.

[tex]\frac { 24 \ in ^2 }{2 \ in }= \frac{ 2 \ in *h}{ 2 \ in}[/tex]

[tex]\frac { 24 \ in ^2 }{2 \ in }= h[/tex]

[tex]12 \ in = h[/tex]

The height of the triangle is 12 inches.

Pamela is 8 years older that Jiri. The sum of their age is 102. What is Jiri's age?

Answers

Answer:

Step-by-step explanation:

102-8= 94

94/2= 47

Jiri is 47 years old.

HLP HLP 10 10 10 HLP HLP HLP

W

Answers

Answer:

A. 6²¹

Step-by-step explanation:

When you have a number raised to the power in that form, you have to multiply the powers:

(6⁷)³

7×3 = 21

(6⁷)³ = 6²¹

Answer:

A. 6 raise to 21

Step-by-step explanation:

its a formula if:

a raise to m whole raise to n = a raise to m×n

so here

          6 raise to 7 × 3

         that is 6 to the power of 21

If the errors produced by a forecasting method for 3 observations are +3, +3, and −3, then what is the mean squared error?

Answers

Answer:

9

Step-by-step explanation:

The mean squared error (MSE)of a set of observations can be calculated using the formula :

(1/n)Σ(Actual values - predicted values)^2

Where n = number of observations

Steps :

Error values of each observation (difference between actual and predicted values) is squared.

Step 2:

The squared values are summed

Step 3:

The summation is the divided by the number of observations

The difference between the actual and predicted values is known as the ERROR.

(1/n)Σ(ERROR)^2

n = 3

Error = +3, +3, - 3

MSE = (1/3)Σ[(3)^2 + (3)^2 + (-3)^2]

MSE = (1/3) × [9 + 9 + 9]

MSE = (1/3) × 27

MSE = 9

divide 111001 by 1101​

Answers

Based on the fact that you asked this three times and got the same answer three times, I suspect the interpretation made by the users that posted those answers was incorrect, and that you meant to ask about dividing in base 2.

We have

111001₂ = 1×2⁵ + 1×2⁴ + 1×2³ + 1×2⁰ = 57

1101₂ = 1×2³ + 1×2² + 1×2⁰ = 13

and 57/13 = (4×13 + 5)/13 = 4 + 5/13.

4 = 2² is already a power of 2, so we have

111001₂/1101₂ = 1×2² + 5/13

we just need to convert 5/13. To do this, we look for consecutive negative powers of 2 that 5/13 falls between, then expand 5/13 as the sum of the smaller power of 2 and some remainder term. For instance,

• 1/4 < 5/13 < 1/2, and

5/13 - 1/4 = (20 - 13)/52= 7/52

so that

5/13 = 1/4 + 7/52

or

5/13 = 1×2 ⁻² + 7/52

Then a partial conversion into base 2 gives us

111001₂/1101₂ = 1×2² + 1×2 ⁻² + 7/52

111001₂/1101₂ = 100.01₂ + 7/52

Continuing in this fashion, we find

• 1/8 < 7/52 < 1/4, and

7/52 = 1/8 + 1/104

==>   111001₂/1101₂ = 100.011₂ + 1/104

• 1/128 < 1/104 < 1/64, and

1/104 = 1/128 + 3/1664

==>   111001₂/1101₂ = 100.0110001₂ + 3/1664

• 1/1024 < 3/1664 < 1/512, and

3/1664 = 1/1024 + 11/13312

==>   111001₂/1101₂ = 100.0110001001₂ + 11/13312

• 1/2048 < 11/13312 < 1/1024, and

11/13312 = 1/2048 + 9/26624

==>   111001₂/1101₂ = 100.01100010011₂ + 9/26624

• 1/4096 < 9/26624 < 1/2048, and

9/26624 = 1/4096 + 5/53248

==>   111001₂/1101₂ = 100.011000100111₂ + 5/53248

and so on.

It turns out that this pattern repeats, so that

[tex]\displaystyle \frac{111001_2}{1101_2} = 100.\overline{011000100111}_2[/tex]

Calculate, correct to one decimal plice
the acute angle between the lines
3x - 4y + 5 = 0 and 2x + 3y -1 = 0
A. 70.69
B. 50.2
C. 39.8
D. 19.4​

Answers

Answer:

A. 70.69 is the correct answer.

Step-by-step explanation:

Given:

Two lines:

[tex]3x - 4y + 5 = 0 \\2x + 3y -1 = 0[/tex]

To find:

Angle between the two lines = ?

Solution:

Acute Angle between two lines can be found by using the below formula:

[tex]tan \theta = |\dfrac{(m_1 - m_2)}{ (1 + m_1m_2)}|[/tex]

Where [tex]\theta[/tex] is the acute angle between two lines.

[tex]m_1, m_2[/tex] are the slopes of two lines.

Slope of a line represented by [tex]ax+by+c=0[/tex] is given as:

[tex]m = -\dfrac{a}{b }[/tex]

So,

[tex]m_1 = -\dfrac{3}{- 4} = \dfrac{3}{4}[/tex]

[tex]m_2 = -\dfrac{2}{ 3}[/tex]

Putting the values in the formula:

[tex]tan \theta = |\dfrac{(\dfrac{3}{4}- (-\dfrac{2}{3}))}{ (1 + \dfrac{3}{4}\times (-\dfrac{2}{3 }))}|\\\Rightarrow tan \theta = |\dfrac{\dfrac{3}{4}+\dfrac{2}{3}}{ (1 -\dfrac{1}{2})}|\\\Rightarrow tan \theta = |\dfrac{\dfrac{17}{12}}{ \dfrac{1}{2}}|\\\Rightarrow tan \theta = \dfrac{17}{6}\\\Rightarrow \theta = tan^{-1}(\frac{17}{6})\\\Rightarrow \theta = \bold{70.69^\circ}[/tex]

So, correct answer is A. 70.69

find the equation of the line that is perpendicular to y=6x-2) and contains to the point (6-,2)

Answers

Answer:

y = -1/6x - 1.

Step-by-step explanation:

I  am assuming that the point id (6, -2).

The slope of the required line = -1/6.

y - y1 = m(x - x1) where m = slope and x1,y1 is a point on the line so we have

y - (-2) = -1/6( x- 6)

y + 2 = -1/6x + 1

y = -1/6x - 1.

trigonometric identities

Answers

Without knowing what Juan's exact steps were, it's hard to say what he did wrong. The least you could say is that his solution is simply not correct.

4 sin²(θ) - 1 = 0

==>   sin²(θ) = 1/4

==>   sin(θ) = ±1/√2

==>   θ = π/4, 3π/4, 5π/4, 7π/4

When csc(Theta)sin(Theta) is simplified, what is the result? StartFraction 1 Over cosecant squared EndFraction StartFraction 1 Over sine squared EndFraction 0 1

Answers

Step-by-step explanation:

csc θ sin θ

(1 / sin θ) sin θ

1

The simplified value of the given expression comes to be 1.

The given expression is:

[tex]cosec\theta.sin\theta[/tex]

What is the trigonometric ratio [tex]cosec\theta[/tex]?

The trigonometric ratio [tex]cosec\theta[/tex] is the ratio of the hypotenuse to the opposite side. It is the inverse of [tex]sin\theta[/tex].

[tex]cosec\theta=\frac{1}{sin\theta}[/tex]

We know that [tex]cosec\theta=\frac{1}{sin\theta}[/tex]

So [tex]cosec\theta.sin\theta[/tex]

[tex]=\frac{1}{sin\theta} .sin\theta[/tex]

=1

So, the simplified value is 1.

Hence, the simplified value of the given expression comes to be 1.

To get more about trigonometric ratios visit:

https://brainly.com/question/24349828

Find the volume of the following figure round your answer to the nearest tenth and if necessary use pi

Answers

Answer:

1526.04

Step-by-step explanation:

the formula for calculating the volume of cone is

V=πr^2(h/3)

Thus,

V = (3.14)(9)^2(18/3)

V = (3.14)(81)(6)

V = 1536.04 yd^3

Rounding off to the nearest tenth, we get

V = 1536 yd^3

Other Questions
hat change is needed to form a complete sentence below? Every year we plant trees near the school. Question 1 options: a) Add a semicolon after Every year b) Change Every to Once a c) Add in the spring after school d) No changes need to be made. Which territory serves as a US Air Force base?Wake IslandNorthern MarianaAmerican SamoaUS Virgin Islands Schoenberg had four compositional phases during his career.A) true B) false Three resistors, each having a resistance, R, are connected in parallel to a 1.50 V battery. If the resistors dissipate a total power of 3.00 W, what is the value of R Explain how mobile phone production could be more sustainable Assuming that the firm is maximizing profits, the marginal cost of the last unit produced equals:________Price Quantity Total cost10 10 809 20 1008 30 1307 40 1706 50 2305 60 300 4 70 380a. $4 b. $40 c. $5 d. $50 e. $6 El documento es importante. (Make it plural Review the list of 15 communication methods below. Select 3 positive and 3 negative methods of communication and explain how the results of each affect your health:Positive 1. 2. 3. Negative 1. 2. 3. I NEED HELP FAST PLEASE Find the mean, median, and mode Please answer this question now In "Name that Player", Do you publish the story as is? Why or why not? Pure competition is important to economists because it is __________________ . Pure competition is important to economists because it is __________________ . the lowest cost to the buyer the benchmark of efficiency to compare all other models against both a. and b. none of the above 2. Would you expect more cooperationor conflict? Community attitudes, zoning restrictions, and quality of labor force are likely to be considered in which of the following location decision methods? a. simulation b. factor-rating method c. transportation method d. locational cost volume analysis e. center-of-gravity method The length and width of a rectangle are measured as 58 cm and 45 cm, respectively, with an error in measurement of at most 0.1 cm in each. Use differentials to estimate the maximum error in the calculated area of the rectangle. 7. Explain why the equation 6|x| + 25 = 15 has no solution.When one solves, they arrive at a step where |x| is equal to a fraction that may not be represented as an integer. Since | x| must be an integer, there is no solution.When one solves, they arrive at a step where x is equal to a negative number. Since x can never be negative inside of the absolute value bars, there is no solution.When one solves, they arrive at a step where |x| is equal to a negative number. Since | x| can never be negative, there is no solution.The statement is false. There is a solution. __________ with age. Question 29 options: The number of hoped-for physical selves increases The number of feared physical selves increases Adults' self-rated health decreases substantially Adults' sense of self-efficacy decreases 50. Carbon mono-oxide can be prepared by heating following substance with Conc. H2SO4. a. Oxalic acid b. Acetic acid c. Ethanoic acid d. All ic 252 QUESTION 9What should be included in the total measurement of output?O a. Formed stools.O b. Perspiration.C. Emesis.O d. Sputum.